USMLE Uworld questions collection (final)
1) A 60-year-old woman is rushed to the emergency room after she started
experiencing a sharp chest pain. Physician examination reveals distant heart
sound and a decrease in systolic pressure during inspiration.
Electrocardiogram shows ST-segment elevation in multiple leads. She has a
history of joint pain but is unsure the exact diagnosis.
This patient most likely has which of the following diagnoses?
A. Osteoarthritis
B. Polymyalgia rheumatic
C. Rheumatoid arthritis
D. Sjogren's syndrome
E. Temporal arteritis
------------------------------------------------------------------------------------
2) A female neonate is born at 36 weeks gestation weighing 7.2 pounds.
Vitals are within normal limits for a newborn. The baby's physical exam is
within normal limits with the exception of a 4/6 heart murmur.
Echocardiography demonstrates ventricular septal defect, and an overriding
aorta that receives blood from both ventricles.
Which of the following cardiovascular changes are also present?
A. Left ventricular hypertrophy and aortic stenosis
B. Left ventricular hypertrophy and pulmonic stenosis
C. Right and left ventricular hypertrophy
D. Right ventricular hypertrophy and aortic stenosis
E. Right ventricular hypertrophy and pulmonic stenosis
, 3) A 20-year-old male presents to the office for a sports physical. He has no
health problems except for prior surgery for lens subluxation. He is 6'4" with
long, thin extremities and digits. His chest has an unusual inward slope, and
he has very flexible joints. Upon cardiac auscultation, an early, soft, diastolic
decrescendo murmur is heard at the apical area. Which of the following is
this patient at the greatest risk for?
A. Aortic dissection
B. Cardiac tamponade
C. Mitral stenosis
D. Myocardial infarction
E. Pulmonary artery hypertension
------------------------------------------------------------------------------------
4) A 50-year-old man presents to the emergency department with severe
substernal chest pain that has not subsided despite taking nitroglycerin. EKG
shows ST-elevations in leads V3-V6. The man promptly treated and is
admitted for observation. One week after the initial chest pain, the patient
again reports having chest pain and the resident physician is attempting to
figure out if this is a complication of the initial attack or a repeat of initial
attack.
Which lab result below would most strongly suggest that the patient is
experiencing a repeat of the initial attack?
A. Elevated brain natriuretic peptide
B. Peaks of troponin I and peaks of lactate dehydrogenase (LDH)
C. Weakly positive CK-MB
D. Weakly positive troponin I
E. Weakly positive troponin I and peaks of lactate dehydrogenase
(LDH)
1) A 60-year-old woman is rushed to the emergency room after she started
experiencing a sharp chest pain. Physician examination reveals distant heart
sound and a decrease in systolic pressure during inspiration.
Electrocardiogram shows ST-segment elevation in multiple leads. She has a
history of joint pain but is unsure the exact diagnosis.
This patient most likely has which of the following diagnoses?
A. Osteoarthritis
B. Polymyalgia rheumatic
C. Rheumatoid arthritis
D. Sjogren's syndrome
E. Temporal arteritis
------------------------------------------------------------------------------------
2) A female neonate is born at 36 weeks gestation weighing 7.2 pounds.
Vitals are within normal limits for a newborn. The baby's physical exam is
within normal limits with the exception of a 4/6 heart murmur.
Echocardiography demonstrates ventricular septal defect, and an overriding
aorta that receives blood from both ventricles.
Which of the following cardiovascular changes are also present?
A. Left ventricular hypertrophy and aortic stenosis
B. Left ventricular hypertrophy and pulmonic stenosis
C. Right and left ventricular hypertrophy
D. Right ventricular hypertrophy and aortic stenosis
E. Right ventricular hypertrophy and pulmonic stenosis
, 3) A 20-year-old male presents to the office for a sports physical. He has no
health problems except for prior surgery for lens subluxation. He is 6'4" with
long, thin extremities and digits. His chest has an unusual inward slope, and
he has very flexible joints. Upon cardiac auscultation, an early, soft, diastolic
decrescendo murmur is heard at the apical area. Which of the following is
this patient at the greatest risk for?
A. Aortic dissection
B. Cardiac tamponade
C. Mitral stenosis
D. Myocardial infarction
E. Pulmonary artery hypertension
------------------------------------------------------------------------------------
4) A 50-year-old man presents to the emergency department with severe
substernal chest pain that has not subsided despite taking nitroglycerin. EKG
shows ST-elevations in leads V3-V6. The man promptly treated and is
admitted for observation. One week after the initial chest pain, the patient
again reports having chest pain and the resident physician is attempting to
figure out if this is a complication of the initial attack or a repeat of initial
attack.
Which lab result below would most strongly suggest that the patient is
experiencing a repeat of the initial attack?
A. Elevated brain natriuretic peptide
B. Peaks of troponin I and peaks of lactate dehydrogenase (LDH)
C. Weakly positive CK-MB
D. Weakly positive troponin I
E. Weakly positive troponin I and peaks of lactate dehydrogenase
(LDH)